Barron’s TOEFL iBT 15th Edition by Pamela J. Sharpe Ph.D.

Barron's TOEFL iBT 15th Edition by Pamela J. Sharpe Ph.D.

Barron’s TOEFL iBT with CD-ROM and MP3 audio CDs, 15th Edition by Pamela J. Sharpe Ph.D.

The most up-to-date TOEFL test prep manual on the market, the fifteenth edition of Barron’s TOEFL iBT is available in options that include a book with two MP3 CDs or book with two MP3 CDs and CD ROM. The manual presents:

  1. A general overview of the TOEFL iBT
  2. Seven model TOEFL iBT tests with answer explanations and sample writing and speaking responses
  3. A review of required academic skills, including note taking, paraphrasing, summarizing, and synthesizing
  4. A review of language skills, including listening, speaking, reading, and writing in English

The manual can be purchased with two MP3 CDs or with a CD-ROM and two MP3 CDs. The CD-ROM presents iBT versions of the book’s 7 TOEFL practice tests plus a bonus TOEFL iBT. The tests on the CD-ROM simulate actual test-taking conditions. The MP3 CDs present audio prompts for the Listening, Speaking, and Writing sections of all TOEFL practice exams and exercises in the manual.

System Requirements:

Microsoft® Windows® PC
2.33GHz or faster x86-compatible processor, or Intel Atom™, 1.6GHz or faster processor for netbook class devices
Microsoft® Windows Server 2008, Windows 7, Windows 8.1 Classic or Windows 10
512MB of RAM (1GB recommended)
Speakers and microphone, or an integrated headset with mic
Plug-Ins: Latest versions of Adobe Air (Included on the CD-ROM)

MAC® OS
Intel® Core™ Duo 1.83GHz or faster processor
Mac OS X v10.7, and above
512MB of RAM (1GB recommended)
Speakers and microphone, or an integrated headset with mic
Plug-Ins: Latest versions of Adobe Air (Included on the CD-ROM)

  TOEFL' s Independent Task; TOEFL' s Integrated Task; university acceptance; Test Preparation; IELTS; TOEFL' s Independent Task; TOEFL' s Integrated Task; university acceptance; Test Preparation; IELTS; TOEFL' s Independent Task; TOEFL' s Integrated Task; university acceptance; Test Preparation; IELTS; TOEFL' s Independent Task; TOEFL' s Integrated Task; university acceptance; Test Preparation; IELTS;
  Writing for the TOEFL iBT Essential Words for the TOEFL, 7th Edition Practice Exercises for the TOEFL with MP3 CD, 8th Edition TOEFL Strategies and Tips with MP3 CDs, 2nd Edition
Product Format Get the guidance you need to succeed on the TOEFL Writing Tasks. Students can test their mastery of TOEFL vocabulary. Preparation for the ITP and the iBT Tests. Detailed, step-by-step coaching designed to help students succeed on the Test of English as a Foreign Language.
Product Highlights This manual prepares students to succeed on the TOEFL’s Independent Task, the all-important essay question, and on the Integrated Task, which combines reading, listening, and writing skills. The author presents a three-step program designed to help students write like native speakers of English. His coaching entails gathering ideas, organizing details, and developing the chosen topic into clear, grammatical written English. He also provides exercises in proofreading and editing. Model essays and model integrated tasks are included for students to read and analyze. Lectures typical of those presented on actual tests are contained on the MP3 CD enclosed with the manual. This revised book is specifically designed for ESL students preparing to take the TOEFL and features updated vocabulary that is seen most often on the exam. It includes: phrases and “purpose” words (for example, define, discuss, claim, etc.) that are used in the speaking and writing sections of the test; new words and phrases, since you are often asked to explain the meaning of a phrase; lists of vocabulary words with definitions, sample sentences, and practice exercises for 500 need-to-know words; detailed advice to help students expand their English language vocabulary, and more. The included practice test with answer key, which is designed to help students evaluate their progress, and increase confidence in their vocabulary skills. This package is designed to prepare students to succeed on both the ITP (Institutional TOEFL Program) and the iBT (Internet Based TOEFL). Updated to reflect the latest TOEFL formats, this book and MP3 CD package presents more than 1,000 practice questions with explanatory answers, as well as example essays and speaking responses. A full-length ITP practice test and a full-length iBT practice test are also included, and are supplemented with instructions for evaluating answers to test questions and determining a test score. The enclosed MP3 CD offers extra practice for the Listening, Speaking, and Integrated Writing sections. This edition includes a bonus chapter with a dictionary of the top 100 academic vocabulary words on the TOEFL, along with ten exercises to test proficiency. An opening Orientation chapter discusses the TOEFL’s five test areas, explains how they are scored, and offers a general test preparation plan. Subsequent chapters set forth detailed strategies and exercises for success in all five test sections of both the iBT and the ITP TOEFL: Reading, Listening, Speaking, Structure, and Writing. Accompanying MP3 CDs complement all of the book’s test strategies. All audio material contained on the compact discs is also presented in printed form in the book’s Audioscripts section. Answer Keys for all questions appear at the back of the book. This book helps students prepare for the TOEFL before they take the test, and it helps them outsmart the TOEFL while they are actually taking the test. The book and MP3 CD program offer valuable TOEFL test preparation when used alone, and it is even more effective when used with Barron’s TOEFL iBT with CD-ROM and MP3 audio CDs, 15th Edition.
         

Table of Contents:

About the Author
Acknowledgments
Sources/Credits
Timetable for the TOEFL® iBT

Barron's TOEFL iBT 15th Edition by Pamela J. Sharpe Ph.D.

Barron’s TOEFL iBT 15th Edition by Pamela J. Sharpe Ph.D.

1. ORIENTATION
Orientation to the TOEFL® iBT
To the Student: How to Use This Book to Succeed
To the Teacher: Recommendations and Resources
Syllabus Options
FAQs―Frequently Asked Questions About the TOEFL® iBT
Study Habits

2. REVIEW OF TOEFL® iBT SECTIONS
Reading
Listening
Speaking
Writing

3. ACADEMIC SKILLS
Campus Vocabulary
Taking Notes
Paraphrasing
Summarizing
Synthesizing

4. MODEL TESTS
Model Test 1: Pretest
Model Test 2: Progress Test
Model Test 3: Progress Test
Model Test 4: Progress Test
Model Test 5: Progress Test
Model Test 6: Progress Test
Model Test 7: Progress Test
Model Test 8: CD-ROM Test

5. ANSWERS AND AUDIO SCRIPTS FOR ACTIVITIES, QUIZZES, AND MODEL TESTS
Answers and Audio Scripts for Quizzes in Chapter 2
Answers and Audio Scripts for Practice Activities in Chapter 3
Explanatory or Example Answers and Audio Scripts for Model Tests
Model Test 1: Pretest
Model Test 2: Progress Test
Model Test 3: Progress Test
Model Test 4: Progress Test
Model Test 5: Progress Test
Model Test 6: Progress Test
Model Test 7: Progress Test
Model Test 8: CD-ROM Test

6. SCORE ESTIMATES
Important Background Information
Procedure for Scoring
Reference Charts
Examples of Scoring Model Tests
Score Comparisons
Feedback
Options for Personal Evaluation

7. RESOURCES
TOEFL Regional Registration Centers
Web Sites for TOEFL
TOEFL® iBT Resource Centers
Glossary of Campus Vocabulary
Barron’s CD-ROM Documentation
Audio CD Tracks

Now you can Free Download Barron’s TOEFL iBT with CD-ROM and MP3 audio CDs, 15th Edition by Pamela J. Sharpe Ph.D. at here: 

Download PDF Book     Audio CD1      Audio CD2

If you can not get the direct download link, Please leave your email below comment & I will send the Ebook + AudioCD for you.

Thank and good luck with TOEFL Exam! 

Click to rate this post!

5 Best TOEFL Books to Boost Your Score Considerably 2018

5 Best TOEFL Books to Boost Your Score Considerably 2018

Do you want to waste your precious money by buying unhelpful books? Since there are literally dozens and dozens of books, it is hard to determine which are the best TOEFL books in the market. To help you choose the best TOEFL books, I will recommend some to boost your scores significantly.

These books have helpful instructions for each section of the test and also includes TOEFL practice tests so you can be ready for the actual test. I have broken down this article into two types of books: books for test preparation and books for TOEFL practice tests. I hope that you will find each breakdown to be helpful as you prepare for this test.

 

Two Types of TOEFL Books

Books for Test Preparation

These books are excellent at teaching different strategies to solve different types of questions. They will explain in-depth of how to solve the questions effectively to guarantee a good TOEFL score. However, their TOEFL practice tests are either too difficult or too easy compared to the actual test.

Books for Practice Tests

These books provide excellent TOEFL practice tests that accurately portray the actual difficulty of the test. The practice books that I will be recommending contains only the authentic practice tests that were previously used as actual tests. However, these books contain little to no explanation on how to solve different types of questions.

I recommend buying at least one strategy book and one practice test book. If you have more money to spend, I recommend buying more practice test books rather than buying more test preparation books so you can solve more authentic practice questions and boost your score. If you buy at least one test preparation book and one practice test book, you will learn strategies from the strategy book and practice your strategies on authentic practice questions.

 

Best TOEFL Books

This are the recommended TOEFL books to help in your test preparation. As I stated earlier, you should buy at least one test preparation book and one TOEFL practice test books to get a good score.

Books for Test Preparation

  1. Barron’s TOEFL iBT with CD-ROM and MP3 audio CDs, 15th Edition
  2. Cambridge Preparation for the TOEFL Test Book with Online Practice Tests

Books for TOEFL Practice Tests

  1. Official TOEFL iBT® Tests Volume 1, 2nd Edition
  2. Official TOEFL iBT® Tests Volume 2
  3. Official Guide to the TOEFL Test With CD-ROM, 4th Edition

 

Best TOEFL Books for Test Preparation

1. Barron’s TOEFL iBT with CD-ROM and MP3 audio CDs, 15th Edition

Barron's TOEFL iBT 15th Edition by Pamela J. Sharpe Ph.D.

Barron’s TOEFL iBT 15th Edition by Pamela J. Sharpe Ph.D.

I highly recommend Barron’s TOEFL book because of the following reasons:

  1. The 150-page strategy guide is very comprehensive.
  2. It contains many practice materials to solidify your test-taking skills.
  3. The audio CDs contains every recording for listening practice questions.
  4. The CD-ROM contains eight practice tests which simulates the actual test taking conditions.
  5. At the back of the book, there are comprehensive explanations for each question on the model tests. This 150-page explanation is very detailed, and you will recognize your weaknesses and strengthen them by using this book.
  6. The cost of this book is inexpensive compared to other TOEFL preparation books.

Barron’s book is one of the best TOEFL books for test preparation. To help you to determine if Barron’s book is right for you, I will breakdown each section of this book.

A. Introduction

This book begins with frequently asked questions about the test, general test-taking tips and advice, how to use this book, and study habits to prepare for TOEFL. These are very informative for new test takers to know what to expect on the test day which will make them confident on the actual test.

 

B. Detailed breakdown of each section and specific strategies for each section

This book does a wonderful job of listing every possible type of questions in each section and checklists that you should follow for each type of questions. For example, the reading section has fourteen possible types of questions such as interference, classification, and insert; the listening section has ten possible types of questions such as purpose, main idea, and connection; the speaking section has six different types of questions such as experiences, preferences, and examples; the writing section has two different types of questions which are synthesis of opposing ideas in integrated essay question and opinion in independent essay.

Not only does the book list every single possible types of questions that could appear on the test, the book gives specific detailed checklists for each possible type of questions. For example, there are about twelve items on checklists for each types of questions. Also, the book lists the point value and the frequency that a certain type of question appears on the TOEFL test.

After the strategy guide, the book enforces those strategies and checklist through many practice problems. The detailed explanation at the back is really helpful because you will know which types of questions are your weaknesses and focus on those weaknesses to strengthen them.

 

C. Academic Section

After Barron’s book has gone over specific strategies for specific sections, it has a section called Academic skills that apply to all four sections.

Campus Vocabulary

Campus vocabulary is a list of approximately 200 academic vocabularies that are frequently used in TOEFL. It includes the word’s definitions, example sentences, and the grammatical use in the sentence. There is also an interactive program which includes all the campus vocabulary in this book. If you are interested in this free program, visit this site.

Taking Notes

The Barron’s book has excellent guides about taking notes and exercises to practice this skill. These excellent strategies teach students to properly take notes such as dividing the paper into columns, separating content into major and minor points, noticing key words and phrases, and using abbreviations and symbols for faster note-taking. Also, the comprehensive lists of keywords, key phrases, abbreviations, and symbols are very helpful.

Paraphrasing

Paraphrasing is rewriting the idea into your own words. This skill is important in all four sections because those sections contain questions about paraphrasing. Skills such as substituting synonyms, using different grammatical structures, and using key phrases help you to improve your paraphrasing skills greatly. After the guide, it has many practice questions to reinforce this skill.

Summarizing

Summarizing is using your own words to shorten an idea into few words. Summarizing is like paraphrasing but using fewer words than paraphrasing. Barron’s book teaches many summarizing skills such as condensing the ideas, combining sentences using clauses of addition, result, contrast, and reversal and identifying the main points. After the guide, it contains questions to strengthen your skills.

Synthesizing

Synthesizing is combining two ideas into one which is important for integrated speaking and writing tasks. This book gives useful advice on synthesizing such as recognizing keywords and phrases that denote relationships and step-by-step guide to reading and writing synthesis tasks.

 

D. 8 TOEFL Practice Tests

Barron’s book contains 7 practice tests and 1 extra test on the CD-ROM which is a total of 8 practice tests. The CD-ROM simulates the all 8 practice tests accurately like the actual tests. Also, in the back, there are two audio CDs for skill-building exercises for each sections.

The test questions in the Barron’s book does not accurately reflect the difficulty of the real TOEFL test. Barron’s questions are little harder than the real TOEFL test which could be beneficial because the actual test will seem easier than the questions in Barron’s. So, do not be discouraged if you miss more questions than you normally would. For books that contain questions with same difficulty as the actual tests, keep on reading!

 

E. Explanation

One aspect that I absolutely adore in the Barron’s Test is the detailed answer explanations of every question in every practice test. The explanation section itself is over 150 pages long! This section contains very detailed explanation of the reason behind each answer. For example, it will state that a certain answer is right because of this specific phrase. Also, it contains sample answers and checklists for every speaking and writing section so you can check your answer. The detailed explanation will help students to recognize their weakness and to strengthen their weakness through constant practice.

 

F. Conclusion

The Barron’s book is extremely helpful because of the thorough strategy guides for each section and the detailed explanation on the back. Although Barron’s book has its flaw of not reflecting the actual difficulty of the test, you will be much more prepared for the actual tests.

Barron’s book should be your first-choice strategy guide because of its thoroughness and its low cost. This book costs around 25 dollars on Amazon which is very cheap compared to other strategy books on the market. 

Free Download this Book at Here: Download this Book

2. Cambridge Preparation for the TOEFL Test Book with Online Practice Tests

Cambridge-Preparation-for-the-TOEFL-Test-Book-

Cambridge University Press is the leading publisher on TOEFL preparation books. This book is the best preparation book on the market (Barron’s close second) because it thoroughly reviews all four sections. It contains many practice skills found in Barron’s and even more.

This book is especially excellent for students who struggle in the writing section. It covers the not only the basics such as developing full paragraph essay and recognizing the main points but also the specifics such as including the specific examples from the text.

There are seven online tests included in this book. The practice tests are the most accurate representation of the test format, not the difficulty of the test. In listening sections, other programs start the clock whenever you start to listen, but on the actual test and in Cambridge, the clock starts running when you started to answer the questions. The Cambridge online tests are the most accurate representation of the test format of the TOEFL and also give your score after each practice tests.

Although the test format itself is the most accurate, the test difficulty and the practice materials are harder than the actual test just like Barron’s. So, don’t be harsh on yourself whenever you have scored lower than your average score.

The detrimental flaw of this book is the high cost. There are two versions of this book: book without audio CDs and book with audio CDs. The book without audio CDs costs around 50 dollars, and the book with CDs costs around 80 dollars. Without the audio CDs, you won’t be able to use half of the skill-building exercises, so the audio CDs are essential to buy. Compared to the cost of Barron’s book which comes with audio CDs, it is extremely expensive.

This book is for students who are aiming to receive 100+ on your TOEFL. If you are not aiming for above 100, then don’t waste money buying this expensive book. Also, if you don’t want to spend much money on practice books, the Barron’s book is an excellent substitute of this book for students who are aiming above 100. However, if you do have the money and willing to spend it, this book is an excellent choice.

Free Download this Book at Here: Download this Book

Best TOEFL Books for TOEFL Practice Tests

The only books that I recommend for TOEFL practice tests are the books from ETS, the company that administers the actual tests. Other companies produce many practice tests, but they are poorly-made, too-easy, or too-difficult. Also, some of those tests have critical errors such as two possible answers to a question. The most authentic practice materials are from ETS which makes the tests and therefore have zero errors. ETS currently offers 13 official tests that were previously administered available for students to purchase. I recommend only these books that contains authentic questions for practice tests.

 

1. Official TOEFL iBT® Tests Volume 1, 2nd Edition

Official TOEFL iBT® Tests Volume 1

Official TOEFL iBT® Tests Volume 1

This book contains five TOEFL practice tests that were previously administered. This book also contains an interactive CD-ROM which is very similar to the real internet test. This is highly recommended for you to get a high score.

Also, the price is relatively inexpensive compared to other books. 

Free Download this Book at Here: Download this Book

2. Official TOEFL iBT® Tests Volume 2

Official TOEFL iBT® Tests Volume 2

Official TOEFL iBT® Tests Volume 2

This book also contains five latest TOEFL practice tests that ETS administered. Like the Volume 1 of the iBT Tests, this book also comes with an interactive CD-ROM which is similar to the real test. 

Free Download this Book at Here: Download this Book

Each of these books contains five complete past TOEFL exams. The TOEFL iBT books are the best to use for practice TOEFLs because they are written by the same people who make the TOEFL and the practice tests included are all retired real TOEFL exams. Like the Official Guide to the TOEFL Test, this means that you can be sure that the practice tests will be very similar in format, style, and content to the actual TOEFL which some unofficial practice tests don’t always succeed in.

Each TOEFL iBT book includes a DVD-ROM so you can take the practice tests either on paper or on the computer. This means you can take practice tests in the same format you’ll take the real TOEFL in. In addition to high-quality practice tests, these TOEFL preparation books also include in-depth answer explanations so you can understand why a particular answer is correct.

Pros:
  • The only prep books with official practice tests (other than the Official Guide to the TOEFL).
  • Option to take the tests either with paper and pencil or on the computer.
  • Clear, easy to understand explanations.
  • The computer versions of the test have a format very similar to the actual iBT TOEFL, so you can get used to how the test will look before exam day.
Cons:
  • These books primarily focus on practice tests and answers and don’t include a lot of strategies or tips for answering questions. If you’re looking for this information, you’ll likely have to purchase another book in addition to these.
  • No sample responses for the Writing or Speaking sections are included.

 

3. Official Guide to the TOEFL Test With CD-ROM, 4th Edition

Official Guide To The TOEFL Test 4 Edition

Official Guide To The TOEFL Test 4 Edition

This book contains three authentic practice tests that were previously administered. Although it has strategies in the beginning, this guide is extremely inferior to the strategy guides in Barron’s or Cambridge. However, it contains interactive CD-ROM to help you familiarize yourself with the real test-taking conditions.

Conclusion

These three books have a total of 13 different authentic tests that were administered previously. 13 tests will be enough to practice the strategies that you have learned using the strategy books.

As I have noted earlier, these books for practice tests contains little or no strategies but is useful for authentic practice tests only. For strategy guides, use Barron’s or Cambridge book.

In another article, I have compiled a list of six free materials for the TOEFL, and you can find it here.

Free Download this Book at Here: Download this Book

 

 

Notice: Now have the latest version of Official Guide to the TOEFl Test – Fifth Edition, You can download free at below link here: The Official Guide to the TOEFL Test with DVD-ROM, Fifth Edition

 

Click to rate this post!

TOEFL IBT Reading Practice Test 20 Solution & Explanation

TOEFL IBT Reading Practice Test 20 Solution & Explanation

Solution & Explanation for TOEFL iBT Reading Practice Test 20 ( From Barron’s TOEFL’S iBT)

 

Reading 1 “The Hydrologic Cycle”TOEFL IBT Reading Practice Test 20 Solution & Explanation

1. B Solar energy is the source of power for the hydrologic cycle paraphrases “The hydrologic cycle is driven by solar energy” and begins by evaporating water from plants, soil, oceans, and freshwater sources paraphrases “evaporates water from oceans, freshwater bodies, soils, and vegetation.”

2. D “The hydrologic cycle is the transfer of water from the oceans to the atmosphere to the land and back to the oceans.” Choices A and B are not correct because they are not complete since they refer to only part of the cycle. Choice C is not correct because it refers to water sources, not the process.

3. D “Of the total 1.3 billion cubic km water on Earth, about 97% is in oceans, and about 2% is in glaciers and ice caps. The rest [of the water] is in freshwater on land and in the atmosphere.” The pronoun phrase “The rest” does not refer to Choices A, B, and C.

4. C “… water on land [freshwater] is important in moving chemicals, sculpturing landscape, weathering rocks, transporting sediments, and providing our water resources.” Choice A is not correct because the rate of evaporation is not compared. Choice B is not correct because 97% of the water is in oceans, not freshwater sources. Choice D is true, but it is not the reason why freshwater is considered important.

5. B “… the building of large dams and reservoirs, can change the amount of water evaporated into the atmosphere and change the location and amount of precipitation on land.” Choice A is not correct because pavement increases flooding. Choice C is not correct because it refers to the purpose of the man-made water sources, not to their effect on the water cycle. Choice D is not correct because aqueducts transport water from the mountains, but they do not improve the flow into the oceans.

6. B In this passage, part is a synonym for “component.” Context comes from the references to “percentages.”

7. C In this passage, basic is a synonym for “fundamental.” Context comes from the usage with “unit,” which is often described as “basic” or “fundamental.”

8. A “A drainage basin is usually named for its main stream or river, such as the Mississippi River drainage basin.” The phrase “such as” signals an example. Choices B, C, and D are true, but they are not the reason that the author mentions the Mississippi River.

9. D “… this relatively small amount of water in the global water cycle {0.001 % of the total water on Earth] . . . produces all our freshwater resources.” Choice A is not correct because the residence time of 9 days is more than one week. Choice B is not correct because both glaciers and oceans are unsuitable for human use. Choice C is not correct because only a relatively small amount of water is in the global water cycle at any one time.

10. C ”… 99% of Earth’s water in its natural state is unavailable or unsuitable for beneficial human use.” Choice A is not correct because total water abundance is not the problem. Choice B is not correct because water can be found at or near the Earth’s surface. Choice D is not correct because the age of water is not mentioned as a safety hazard.

11. C In this passage, important is a synonym for “significant.” Context comes from the numbers in the fraction.

12. C Cause and effect is a transitional device that connects the insert sentence with the previous sentence. The cause is “distribution of water on land is far from uniform” and the result is water shortages in some areas. Choices A, B, and D are not correct because the cause and result are not in consecutive order.

13. C, E, A summarize the passage. Choice B is a minor point that refers to major point C. Choice D can be inferred from the passage, but it is not developed as a major point. Choice F is true but it is not mentioned in the passage.

Reading 2 “Piaget’s Cognitive Development Theory”

14. C We engage in both organization of what we see and experience paraphrases”… In addition to organizing our observations and experiences” and adaptation of novel ideas paraphrases “we adapt, adjusting to new environmental demands.”

15. A The concepts are explained by the toddler’s new experience with the car as the child “has assimilated these objects . .. and fine-tunes the category . . . accommodating the scheme.” Choice B is not correct because the concepts, not the demonstration, are the lesson. Choice C is not correct because the toddler solves the problem. Choice D is not correct because the example demonstrates the ways that people adapt, not the stages of development.

16. A In this passage, change is a synonym for “adjust.” Context comes from the reference to “adapt” in a previous sentence.

17. B “Some objects such as fingers and the mother’s breast, can be sucked, but others [other objects], such as fuzzy blankets, should not be sucked.” The noun “others” does not refer to Choices A, C, and D.

18. C In this passage, different is a synonym for “distinct.” Context comes from the reference to “different” in the next sentence.

19. B In this passage, complex is a synonym for “sophisticated.”

20. B “… operations, which are internalized mental actions.” Choice A is not correct because symbolic thought occurs in a later stage, after operations. Choice C is not correct because it occurs in an earlier stage, before operations. Choice D is not correct because the reasoning that children can perform in operational stages does not explain the term operations.

21. B “. . . concrete operational thinkers cannot imagine the steps necessary to complete an algebraic equation, which is too abstract for thinking at this stage.” Choice A is not correct because algebra requires formal, not concrete, operational thinking. Choice C is not correct because a child of 10 has reasoning abilities, if they are applied to concrete examples. Choice D is not correct because it is the abstract nature of the steps, not the number of steps, that makes algebra too difficult for a 10-year old.

22. C “They might think about what an ideal parent is like and compare their parents to this ideal standard.” Choice A is true, but it is not the reason that the author mentions parents. Choices B and D are not mentioned in the passage.

23. A Because the formal operational stage is the last stage in Piaget’s theory, and the age range is between 11 and 15, it must be concluded that people who are older than 15 have completed all of the stages. Choice B is not correct because the age range for the formal operational stage is between 11 and 15. Choice C is not correct because logical reasoning replaces intuitive thought in the concrete operational stage from 7 to 11 years of age. Choice D is not correct because there is no evidence to support this conclusion in the passage.

24. C Choice A is mentioned in paragraph 4, sentence 2. Choice B is mentioned in paragraph 3, sentences 2 and 3. Choice D is mentioned in paragraph 7, sentences 3 and 4.

25. C Chronological order is a transitional device that connects the insert sentence with the following sentence. “At the beginning” should appear in the sentence before “At the end” in reference to the sensorimotor stage.

26. E, A, B summarize the passage. Choice C is an explanation of the concrete operational stage, which is a minor point that is used to develop major point B. Choice D is an example of accommodation that supports major point A. Choice F is true, but it is not directly referred to in the passage.

Reading 3 “Conquest by Patents”

27. D “… patents are about the control of technology.” Choices A and B are not correct because protests and lawsuits are caused by patents, but they are not the reason for patents to exist. Choice C is not correct because the “incentive and reward” to inventors is the reason touted [publicized] but not the real reason.

28. D “In the 1760s … Arkwright invented the water-powered spinning frame, a machine destined to bring cotton-spinning out of the home and into the factory. It [a machine] was an invention.” The pronoun “If does not refer to Choices A, B, or C.

29. A Among the laws to protect Britain from competition paraphrases “To protect its [Britain’s] competitive advantage . . . Parliament enacted a series of restrictive measures,” and the textile industry paraphrases “manufactured cloth.”… a ban on exporting Arkwright equipment paraphrases “the prohibition of the export of Arkwright machinery” and the [ban on] emigration of former employees paraphrases “the emigration of any workers who had worked in factories using it [Arkwright machinery].”

30. C “… Samuel Slater, who had worked for years in the Arkwright mills, left England … disguised as a farmer… he was an intellectual property thief.” Choice A is not correct because Slater established the textile mill in America, not in Great Britain. Choice B is not correct because Slater was the only worker from Britain. Choice D is not correct because Slater broke the law.

31. A In this passage, discoveries is a synonym for “innovations.” Context comes from the introduction in the first paragraph that explains the “rights” for “inventors.”

32. B “By the early 1970s… U.S. industry demanded greater protection for its idea-based products.”

Choice A is not correct because the free exchange was favored earlier in the history of the United States. Choice C is not correct because the United States pushed for standards in international trade agreements. Choice D is not correct because the United States blamed the Third World nations for piracy.

33. A “… a counterclaim that these were ‘natural’ or ‘raw* materials and therefore did not qualify for patents.” Choice B is not correct because a high percentage Of the materials originated in plant and animal germplasm taken from the developing world. Choice C is not correct because barely a cent of royalties had been paid. Choice D is a claim against pirates in the Third World, but it is not a justification for using plants and animals from the developing world.

34. B “Such unacknowledged and uncompensated appropriation they named ‘biopiracy.’” Choices A, C, and D are not part of the author’s definition.

35. B In this passage, assist is a synonym for “facilitate.” Context comes from logical reasoning in the sentence that suggests a positive effect. Choice D can be eliminated because it would have a negative effect.

36. D In this passage, ideas is a synonym for “notions.” Context comes from the reference to the abstract concepts of “property and creativity.”

37. A The word “Conquest” conveys the idea of domination, power, and unfair practices. Choices B and C are true, but they do not explain the use of the word “Conquest.” Choice D is not correct because the trade agreements prevent developing nations from exerting the power that they might obtain through ownership of valuable resources.

38. A Chronological order as well as cause and effect are transitional devices that connect the insert sentence with the previous and following sentences. “Arriving in the U.S.” in the insert sentence would have to follow the reference in the previous sentence to the time when Slater “left England for the New World.” The recreation of the factory in the insert sentence was the cause that “enabled the production of commercial-grade cotton cloth in the New World” mentioned in the following sentence.

39. A, D, F summarize the passage. Choices B and C provide an example that develops major point A.

Choice E is a detail that supports major point D.

Click to rate this post!

TOEFL IBT Reading Practice Test 19 Solution & Explanation

TOEFL IBT Reading Practice Test 19 Solution & Explanation

Solution & Explanation for TOEFL iBT Reading Practice Test 19 ( From Barron’s TOEFL’S iBT)

Reading 1 “Exam and Endangered Species”

1. C . exotic species … a resident of an established community that was deliberately or accidentally moved from its home range and became established elsewhere.” Choice A is not correct because it refers to an endangered species, not an exotic species. Choice B is not correct because exotic species are moved from their communities. Choice D is not correct because an exotic species becomes established, unlike most imports, which fail to thrive outside of their home range.

2. D “Unlike most imports, which can’t take hold outside their home range, an exotic species permanently insinuates itself [the exotic species] into a new community.” The pronoun “itself does not refer to Choices A, B, or C.

3. B In this passage, connect is a synonym for “bond.” Context comes from the result at the end of the same sentence. “… they started to import familiar animals.”

4. C “… no natural predators… was the reality.” Choice A is not correct because it refers to a solution for the problem, not why the plan failed. Choice B is not correct because Australians imported rabbits because they liked the familiar species. Choice D is not correct because it refers to the reason that the rabbits were introduced, not to why the plan failed.

5. C The rabbits create deserts by eating the vegetation, but they were not moved to deserts.

Choice A is mentioned in paragraph 4, sentence 6. Choice B is mentioned in paragraph 4, sentence 7. Choice D is mentioned in paragraph 4, sentence 9.

6. B “Biting insects, mainly mosquitoes and fleas, quickly transmit the virus from host to host.”

Choice A is not correct because South American rabbits are the normal hosts for the myxoma virus. Choice C is not correct because it is the O. cuniculus rabbit that dies when infected. Choice D is not correct because resistant populations of O. cuniculus rabbits, not fleas, have an immunity to the virus.

7. C “… researchers are now questioning whether… it can … infect animals other than rabbits (such as humans).” Choice A is not correct because insects were not mentioned in the Spencer Gulf experiment. Choice B is not correct because the purpose of the experiment was to kill the rabbits. Choice D is not correct because 80 to 95 percent of the rabbits are being killed, but the small number with immunity is not identified as dangerous.

8. C In this passage, results is a synonym for “consequences.” Context comes from the logical connection between researchers “questioning” and the phrase “long-term,” which describes the “consequences.”

9. D The farm and processing plant will manufacture products from kudzu, which will offer partial solutions. Choice A is not correct because kudzu was imported to control erosion, not for manufacture. Choice B is not correct because no argument is presented in defense of the decision. Choice C is not correct because it grows in Asia already.

10. A In this passage, surpasses is a synonym for “exceeds.” Context comes from the logical relationship between “demand” and “supply.”

11. D “When you hear someone bubbling enthusiastically about an exotic species, you can safely bet the speaker isn’t an ecologist… they [exotic species] make native species endangered species.” This introduction establishes the author’s opinion that exotic species are often disruptive to the ecology.

12. C Vocabulary reference is a transitional device that connects the insert sentence with the previous sentence. The connection is the reference to “starch” in both the insert sentence and the previous sentence.

13. E, B, C summarize the passage. Choice A is a minor point that refers to major point C. Choice D is a detail that is not developed as a major point. Choice F is an important fact, but it is not a major point because is it not developed.

Reading 2 “Paleolithic Art”

14. C . the remoteness and difficulty of access … suggests]… magical properties … rituals or dances.” Choice A is not correct because they were probably used for rituals. Choices B and D are not mentioned or implied in the passage.

15. D Choice A is mentioned in paragraph 1, sentence 5. Choice B is mentioned in paragraph 1, sentence 6. Choice C is implied in paragraph 1, sentences 3 and 5.

16. A In this passage, admissionIs a synonym for “access.” Context comes from the contrast with “remoteness” and “difficulty” in the same sentence.

17. C In this passage, assist is a synonym for “facilitate.” Context comes from the contrast of “destruction” and “survival” in the same and following sentences.

18. C “A central problem for both the … theories is that the animals that seem to have been diet staples of Old Stone Age peoples are not those [animals] most frequently portrayed.” The pronoun “those” does not refer to Choices A, B, or D.

19. B In this passage, the phrase not believed describes “discredited.” Choice A describes distracted. Choice C describes discouraged. Choice D describes disorderly. Context comes from the parts of the word. The prefix dis means “not.” The root credit means “believe.”

20. A It is true paraphrases “cannot… be doubted” and the paintings were meaningful paraphrases “the paintings did have meaning.”

21. B “… and composition (how the motifs are arranged on the surface)____A definition appears within the parentheses that follow the word. Choices A and C are not correct because neither an example nor a contrasting statement is included in reference to the word composition. Although “art historians” are mentioned in a previous sentence about the intent of the paintings, they are not referred to in order to define a word.

22. B “Some researchers have considered them [positive imprints] ‘signatures* of cult or community members, or… individual artists.” Choices A and C are not correct because they are not mentioned or implied. Choice D is not correct because the author states that the “handprints . . . must have had a purpose.”

23. C “Old Stone Age painters and sculptors frequently . . . used the caves* naturally irregular surfaces to help give the illusion of real presence to their forms.” Choice A is not correct because the hardness of the stone is not mentioned. Choice B is not correct because the rock formation, not the animals, provided inspiration. Choice D may have been true, but it is not the reason that they selected certain surfaces in the caves.

24. C Because the author presents several different theories and does not offer a strong argument for any of them, the author’s opinion is probably that the exact purpose of cave paintings is not known. Choice A is not correct because the author also presents the food-creation theory and the mythology theory as alternatives to the hunting ritual theory. Choice B is not correct because the mythology theory is not the only possibility discussed. Choice D is not correct because the author suggests several reasons why this theory cannot explain the narrow range of subjects or the inaccessibility of many of the paintings.

25. B Example is a transitional device that connects the insert sentence with the general statement in the previous sentence. The connection is between the general statement “animals that seem to have been diet staples . . . are not. . . portrayed” and the example that “red deer, not bison were eaten.”

26. B, D, A summarize the passage. Choice C is true, but it is a minor point that is mentioned as evidence for Choice B. Choice E is not mentioned in the passage. Choice F is true, but it is a point that is used to develop the ideas in Choice A.

Reading 3 “Group Decision Making”

27. A “Groups accumulate more information, knowledge and facts … and often consider more alternatives.” Choice B is not correct because a group tends to view a problem in more than one way. Choice C is not correct because making recommendations instead of decisions is not mentioned or implied in the passage. Choice D is not correct because each person has experience, but the experience of a group is not mentioned as a reason why a group is chosen to participate.

28. B . people will … be more committed to a decision in which they have had a say than to a decision made for them.” Choice A is true, but more ideas do not explain why the decisions are successful. Choice C is not correct because the help provided by a large number of people is not mentioned in the passage as an advantage during implementation. Choice D is not correct because implementation is successful in group decisions, but the decisions themselves may or may not be successful.

29. B In this passage, significant is a synonym for “considerable.” Context comes from the reference to the “time required to make a decision” as a “disadvantage.”

30. D In this passage, As a result describes “Consequently.” Context comes from the conclusion that follows the word “Consequently.”

31. B “One obvious disadvantage of group decision making is the time required to make a decision.”

Choice A is not correct because the implication is that sometimes a decision could have been made as effectively by an individual. Choice C is not correct because the “cost” refers to the time, not to the pay for group members. Choice D is not correct because groups tend to avoid disagreements.

32. B “All group members need to be encouraged and permitted to contribute.” Choice A is not correct because the group should have goals, and personal goals by one member [the leader] should not dominate the discussion. Choice C is not correct because it is considered a disadvantage when an individual such as the group leader dominates the group. Choice D is not correct because expectations are not mentioned as a responsibility of the group leader.

33. B In this passage, debatable is a synonym for “controversial.” Context comes from the contrast with “social pressure … to conform.”

34. C “. .. a group may try too hard to compromise … to maintain friendships and avoid disagreements.” Choice A is not correct because the group may not make optimal decisions when the members try too hard to compromise. Choice B is not correct because groupthink requires agreement rather than compromise. Choice D is not correct because helping one member to reach a personal goal or win an argument would be the opposite of compromise.

35. A “It occurs when groups are highly cohesive, have highly directive leaders, are insulated so they have no dear ways to get objective information, and—because they lack outside information— have little hope that a better solution might be found than the one [solution] proposed by the leader or other influential group members.” The phrase “the one” does not refer to Choices B, C, or D.

36. B “. . . self-appointed ‘mind guards’ . . . bring pressure on dissenters.” Choice A is not correct because people who conform will not necessarily pressure others. Choice C is not correct because “mind guards” use force to exert influence and may not be the most ethical members. Choice D is not correct because “mind guards” do not disagree with the group.

37. C “… decisions … are made without consideration of. .. alternative options.” Choice A is not correct because the group exerts pressure on dissenters, but dissenters do not exert pressure on the group. Choice B is not correct because it is neither mentioned nor implied in the passage. Choice D is not correct because when groupthink takes place, poor decision making and wrong decisions occur.

38. A Generalization and example is a transitional device that connects the insert sentence with following sentences. “In fact, the traditional group is prone to a variety of difficulties” provides a general statement that introduces the disadvantages developed in the following sentences. Choices B, C, and D would interrupt the examples by inserting the generalization.

39. Advantages: C, D, H Disadvantages: A, B, E, F Not used: G, I

next –> Answers for Reading 5 + Reading 6

Click to rate this post!

TOEFL IBT Reading Practice Test 18 Solution & Explanation

TOEFL IBT Reading Practice Test 18 Solution & Explanation

Solution & Explanation for TOEFL iBT Reading Practice Test 18 ( From Barron’s TOEFL’S iBT)

Reading 1 “Rising Sea Levels”TOEFL IBT Reading Practice Test 18 Solution & Explanation

1. B In this passage, scale is a synonym for “range.” Context comes from the numbers in the following sentence, .. 10-20 cm____”

2. B In this passage, probably is a synonym for “likely.”

3. B The author mentions an iceberg “the size of Delaware” to impress the reader with the size of the icebergs. Choices A, C, and D are not noted in the reference to Delaware.

4. D “In response to the increasing warmth, the Antarctic Peninsula is sporting new vegetation growth.” Choices A and C are true, but they are not the reason for the new plant life. Choice B is not correct because the islands have appeared because of ice melt, not because the land masses have split.

5. B “About 8000 km of ice shelf are gone, changing maps, freeing up islands to circumnavigation, and creating thousands of icebergs.” Choice A is not correct because a rise in temperature breaks an ice shelf into icebergs. Choice C is not correct because the reference to islands relates to warmer temperatures and melting ice. Choice D is not correct because mountain glaciers that melt will cause a rise in sea level, not the creation of icebergs.

6. B Most of paragraph 5 is in quotation marks in reference to data in a study by the IPCC. Choice A is not correct because the author is quoting data from a source. Choice C is not correct because data on sea levels does not compare one area with another. Choice D is not correct because the author does not mention his studies when he is presenting the data.

7. A In this passage, definite is a synonym for “conclusive.” Context comes from the phrase “the clearest and best evidence” in the following sentence.

8. A The IPCC did not have the data paraphrases “… the new measurements… did not reach the IPCC.”

9. C “… people move away from coastal flooding from the sea-level rise.” Choice A is not correct

because the temperature on land is not mentioned. Choice B is not correct because the vegetation along the coastlines will die as seawater floods it. Choice D is true, but it is not the reason why people will migrate.

10. C The reference to “Particularly tragic social and economic consequences” in the last paragraph

gives an insight into the author’s opinion. Choices A and B are not correct because they are not directly expressed and cannot be concluded from information in the passage. Choice D is not correct because the breaking up of ice shelves and the melting of ice causes the sea levels to rise, not the new glaciers that are created when the ice shelves disintegrate.

11. D … small island states … among the impacts there [in the states].” Choices A, B, and C refer

to consequences, not to the location.

12. B Alphabetical order is a transitional device that connects the sentences. Larsen A, B, and C are

mentioned in order. The insert sentence is an introductory statement.

13. C, B, E summarize the passage. Choice A is an example that supports major point C. Choice D is a detail that supports major point B. Choice F is an example that supports major point C.

Reading 2 “Organic Architecture”

14. C “One of the most striking personalities in the development of early-twentieth-century architecture was Frank Lloyd Wright____Wright set out to create ‘architecture of democracy.’” Choices

A, B, and D are major points that support the main idea, the work of Frank Lloyd Wright.

15. C According to a quote by Wright, “This ideal [organic unity]… I called … continuity.” Choice A is not correct because it refers to classical architecture, not to organic architecture. Choice B is not correct because, although he considered organic architecture his ideal, he referred to it as “continuity.” Choice D is not correct because Wright rejected classic architecture.

16. A “Wright manifested his vigorous originality early, and by 1900 he had arrived at a style entirely his own [style].” The phrase “his own” does not refer to Choices B, C, or D.

17. C In this passage, created is a synonym for “conceived.” Context comes from the word “designed” in the previous sentence.

18. D In this passage, Discontinuing is a synonym for “Abandoning.” Context comes from the contrast of “symmetry” with the “spaces… grouped freely.”

19. B “Wright fully expressed these elements and concepts in Robie House.” Choice A is true but it is only orie of the original ideas expressed. Choice C is not correct because, although it is often true of Wright1 s designs, the accessories for Robie House were not mentioned in the passage. Choice D is not correct because the house was built between 1907 and 1909, but Wright did not live there during the construction.

20. A In this passage, the phrase most important describes “prime.” The word “prime” is often used with “example.”

21. C Because “the city lot constrained the building-to-site relationship” at “Robie House,” the

description of “Fallingwater” implies that it was better suited to the site. Choice A is not correct because “Fallingwater” was built after “Robie House.” Choice B is not correct because “Falling-water” extended the “Robie House” design in all four directions. Choice D is not correct because “Robie House” had many large and open spaces in the “wandering” floor plan.

22. B “.. . he acted on a cherished dream to provide good architectural design for less prosperous

people by adapting the ideas of his prairie house to plans for smaller, less expensive dwellings.” Choice A is not correct because the smaller, prairie houses were not designed specifically for Europe. Choice C is not correct because many younger architects adopted his designs, but he did not build prairie houses to help architects. The revolution in architecture mentioned in Choice D occurred, but it was not Wright’s purpose in building the smaller versions of his prairie designs.

23. A “The publication of Wright1 s plans brought him a measure of fame in Europe… and an exhibition of his designs [in Berlin]. . . stimulated younger architects to adopt some of his ideas.” Choice B is not correct because lectures are not mentioned in the passage. Choices C and D are correct, but they are not the reasons that Wright’s work became well known in Europe.

24. C The entrance in a prairie house was “all but concealed.” Choice A is mentioned in paragraph 3, sentence 5. Choice B is mentioned in paragraph 3, sentence 7. Choice D is mentioned in paragraph 3, sentence 7.

25. B A general statement followed by examples is’a transitional device that connects the insert sentence with the following sentences. The insert sentence introduces the examples of the relationship between interior and exterior spaces.

26. C, F, D summarize the passage. Choices A, B, and E are details of Wrighfs professional life that are not main points in the passage.

Reading 3 “New Women of the Ice Age”

27. B “Amassing critical and previously overlooked evidence . . . Softer [proposes] a technique of hunting previously invisible in the archaeological evidence.” Choice A is not correct because her theories “make her conservative colleagues cringe.” Choice C is not correct because conservative researchers concentrated on studies of tools and bones and used them to support their theories. Choice D is not correct because she uses not only modem cultural evidence but also archeological evidence, including small game bones and tools that could have been used to make nets.

28. B “Amassing … evidence from Dolni Vestonice and the neighboring site of Pavlov, researchers… propose that human survival there had little to do with men hurling spears at big game animals. Instead … it [survival] depended largely on women, plants, and a technique of hunting previously invisible in the archeological evidence.” The pronoun “it” does not refer to Choices A, C, or D.

29. C In this passage, suggestions is a synonym for “implications.” Context comes from the clause “they raise serious questions” in the same sentence-

30. B Because her views “make her conservative colleagues cringe,” it must be concluded that her views are not as conservative. Choice A is not correct because she disagrees about the role of women as hunters, not as caretakers. Choice C is not correct because she is identified as an authority on the Ice Age and as an archeologist, not as a biologist. Choice D is not correct because she is a leading authority on hunting and gathering in the Ice Age.

31. A In this passage, limit is a synonym for “constrain.” Context comes from the logical supposition in the same sentence.

32. D … historically paraphrases “In many historical societies” and not perilous paraphrases “nor… in physical peril.” …did not require great strength paraphrases “did not call for brute strength,” and women have been important participants paraphrases “women played a key part.”

33. D “‘Everybody and their mother could participate [in net hunting].’” Choice A is not correct because the Australian hunters were not fishermen, and there is no evidence that they developed it. Choice B is not correct because net hunting did not place young mothers in physical peril. Choice C is not correct because net hunting was used to capture game, not to protect the camp.

34. A The author mentions Native American and Aborigine groups to give examples of modem groups in which women participate in net hunting. Choice B is not correct because the author presents facts, not opinions. Choice C is not correct because the techniques do not place women in peril but they do not protect them, either. Choice D is not correct because the example of the people in the Congo reinforces the information about the Native American and Aborigine groups.

35. C “. . . Mbuti in the forests of the Congo report that they capture game every time they lay out their woven traps, scooping up 50 percent of the animals encountered.” Choice A is not correct because nets are valued more than bows and arrows. Choice B is not correct because they trade the surplus meat with neighbors. Choice D is not correct because vegetables are not mentioned.

36. B “… some of their inhabitants whittled bone tools that look much like the awls and net spacers…” Choice A is true but it is not mentioned as evidence. Choice C is not correct because Softer believes that net hunting was more widespread than these two sites. Choice D is not correct because the camps stretched from Germany to Russia, but the researchers from those areas were not mentioned.

37. D In this passage, functions is a synonym for “roles.” Context comes from the reference to “activities” in the same sentence.

38. C A conclusion based on evidence is a transitional device that connects the conclusion in the insert sentence to the evidence in the previous sentences. Choices A and B are not correct because the conclusion in the insert sentence would appear before the evidence. Choice D is not correct because the insert sentence would interrupt the relationship between the first and second sentences in the last paragraph.

39. B, D, F summarize the passage. Choice A is a minor point that is not developed. Choice C is a detail that is mentioned to support major point D. Choice E is true, but it is not mentioned in the passage.

Click to rate this post!

TOEFL IBT Reading Practice Test 17 Solution & Explanation

Solution for TOEFL iBT Reading Test 17

Reading 1 “Layers of Social Class”

1. B

“The different groups are arrayed along a continuum with those [classes] with the most money, education, and prestige at the top.” The pronoun “those” does not refer to choices A, C, or D.

2. A

In this passage, very large describes “enormous.” “… a very small proportion of people” contrasts with very large control. “… vast amounts of wealth” also provides context.

3. A

Although it is not generally accepted paraphrases “Despite social myths to the contrary.”. .. your family provides the best prediction of your future wealth paraphrases”… the best predictor of future wealth is the family into which you are bom.”

4. B

The author uses the example of the Forbes 400 to support the statement that most wealthy people inherit their money. “… most [wealth] is inherited.”

5. B

In this passage, expensive is a synonym for “exclusive.” Context comes from the previous reference to “high-priced.”

6. B

“The upper middle class . . . tend to be well-educated professionals or business executives.”

Choice A is not correct because the lower middle class is composed of workers in skilled trades. Choice C is not correct because the term nouveau riche refers to the upper class that has recently acquired money. Choice D is not correct because the upper class is typically a group that has inherited wealth.

7. B

“.. . an open-class system leads many to think that the majority have a middle-class lifestyle….” Choice A is not correct because they vary widely in lifestyle and in resources. Choice C is not correct because, although the status may be unclear, people do not have a problem defining themselves as middle class. Choice D is not correct because “norm” refers to average, not normal.

8. A

In this passage, mostly is a synonym for “primarily.” Context comes from the root prime, which means “first.”

9. C

“Also known as the working class, this class indues blue collar workers … and many service workers____” Choice A is not correct because they are working in blue collar and service jobs.

Choice B is not correct because service workers and manual laborers are part of the lower middle class, not the lower class that work in minimum-wage jobs. Choice D is not correct because the working class is in the lower middle class, not the lower class.

10. C

“The underclass includes those who are likely to be permanently unemployed … the under class may become dependent on public assistance or illegal activities.” Choices A and D may be true, but they are not mentioned in the passage. Choice B is not correct because the illegal activities cause the increase in crime, not the opposite.

11. D

Choices A, B, and C are mentioned for the first time in paragraph 1, sentence 1.

12.

A Generalization and example is a transitional device that connects the insert sentence with the following sentence. “The working poor constitute a large portion of those who are poor” provides a general statement which is followed by examples, “People of color and women … the working poor. . . .” Choices B, C, and D are not correct because they are not examples of the generalization.

13. C, E, B summarize the passage.

Choice A is a minor point that supports the major point in Choice C. Choice D is a minor point that supports the major point in Choice E. Choice F is true, but it is not developed as a major point.

Reading 2 “Weather and Chaotic Systems”

14. A “Many chaotic systems [like weather] have a kind of underlying order that explains the general features of their behavior____” Choice B is not correct because it was an incorrect assumption.

Choice C is not correct because it was also an incorrect assumption. Choice D is not correct because today we have a very good understanding of the physical laws of atoms.

15. B In this passage, specific is a synonym for “particular.”

16. B In this passage, control is a synonym for “govern.”

17. A The author uses the example of the car to explain how conditions are used to make predictions. The prediction of the location of the car is compared with the prediction of the weather.

18. D “For tomorrow’s weather, this slightly different initial condition will not change the weather prediction … But for next month’s weather, the two predictions may not agree at all!” Choice A is not correct because the change in the initial conditions was minor. Choice B is not correct because it is not mentioned in the passage. Choice C is not correct because computer models are used to deal with all the data for weather prediction on time scales shorter than a few weeks.

19. C . chaotic systems are described by nonlinear equations.” Choice A is not correct because chaotic systems [like weather] “are not completely random.” Choice B is true, but it is not the reason why weather is considered a chaotic system. Choice D is not correct because many chaotic systems are “ ‘predictably unpredictable.’ ”

20. A “This extreme sensitivity to initial conditions is sometimes called the butterfly effect” Choice B is not correct because the flap of a butterfly’s wings is used to describe the sensitivity, not to predict conditions. Choice C is not correct because the rate of the wings is not mentioned. Choice D is not correct because the cause and result do not refer to different locations. They refer to changes over time.

21. D “Simple systems are described by linear equations in which, [in the linear equations] for example, increasing a cause produces a proportional increase in an effect.” The phrase “in which” does not refer to Choices A, B, or C.

22. B The author mentions the economy to provide an example of another chaotic system. “For example, the economy is nonlinear because a rise in interest rates does not automatically produce a corresponding change in consumer spending.”

23. C In this passage, characteristics is a synonym for “features.” Context comes from the contrast with “details” later in the sentence.

24. D “Our understanding of chaotic systems is increasing at a tremendous rate, but much remains to be learned about them.” Choice A is not correct because “many chaotic systems have a kind of underlying order.” Choice B is not correct because “Our understanding of chaotic systems is increasing at a tremendous rate.” Choice C is not correct because “details .. . remain unpredictable.”

25. B Chronological order is a transitional device that connects the insert sentence in sequence within the text. “… tomorrow” should precede “next week” and “next month” should follow “next week.”

26. D, F, C summarize the passage. Choice A may be true, but it is not directly stated in the passage.

Choice B is a minor point because it is an example. Choice E is a minor point because it is an example.

Reading Passage 3 + 4

Click to rate this post!

TOEFL IBT Reading Practice Test 16 Solution & Explanation

TOEFL IBT Reading Practice Test 16 Solution & Explanation

Solution & Explanation for TOEFL iBT Reading Practice Test 16 ( From Barron’s TOEFL’S iBT)

Reading 1 “Symbiotic Relationships”

1. C In this passage, obtains is a synonym for “derives.” Context comes from the close association

with the word “benefit.”

2. C “Parasitism is a relationship in which one organism, known as the parasite, lives in or on

another organism, known as the host, from which it [the parasite] derives nourishment.” The pronoun “it” does not refer to Choices A, B, or D.

3. A In this passage, comparatively is a synonym for “relatively.” Context comes from the phrase

“not killed immediately” in the same sentence.

4. C … when the parasite first invades paraphrases “Newly established” and the most destructive

phase paraphrases “more destructive.”

5. A In this passage, permit is a synonym for “tolerate.” Context comes from the phrase “reducing

the harm” in the same sentence.

6. D “Parasites that live on the surface of their hosts are known as ectoparasites.” . . . surface

means “outside.” Choice A is not correct because “mold and mildew” are ectoparasites, not ways that ectoparasites survive. Choices B and C refer to endoparasites, not to ectoparasites.

7. A “Many examples of commensal relationships exist. Orchids use trees as a surface upon which

to grow.” Choices B and D are mentioned as examples of parasites. Choice C is not correct because ants are mentioned with the Acacia as an example of mutualism, not a commensal relationship.

8. C In this passage, really is a synonym for “actually.”

9. B The author uses the example of the Acacia to explain how two species can benefit from contact. “Some species of Acacia, a thorny tree, provide food in the form of sugar. . . Certain species of ants . . . receive food and a place to live, and the [Acacia] tree is protected from animals… ” Both organisms benefit.

10. C “The bacteria do not cause disease but provide the plants with nitrogen-containing molecules.”

Choice A is not correct because the bacteria do not cause disease. Choice B is not correct because the bacteria is used for growth. Choice D is not correct because the nodules supply nitrogen, which is beneficial.

11. A Pronoun reference is a transitional device that connects the insert sentence with the previous

sentence. “Fleas, lice and some molds and mildews are examples of ectoparasites. They [fleas, lice and some molds] live on the feathers of birds or the fur of animals.” Choices B, C, and D are not correct because the pronoun does not refer to endoparasites, which live inside the host and are explained later in the passage.

12. A Because the text explains the types of relationships between species, it must be concluded

that the passage would be found in a chapter about the environment and organisms. Choices B, C, and D are not correct because they are not as closely associated with the main idea.

13. A, D, E summarize the passage. Choice B is true, but it is an example of a commensal relationship

described in major point D. Choice C refers to a mutualistic relationship developed in major point E. Choice F is mentioned as a minor point in the discussion of parasitic relationships described in major point A.

Reading 2 “Civilization”

14. B “… Neolithic settlements were hardly more than villages. But as their inhabitants mastered the art of farming, more complex human societies [civilizations] emerged.” Choice A is not correct because the Neolithic settlements preceded civilizations. Choice C is not correct because agriculture is mentioned as a cause of the rise in complex cultures, not as a definition of civilization. Choice D is not correct because the population centers increased in size as civilizations grew, but other basic characteristics had to be present as well.

15. B “Although copper was the first metal to be utilized in producing tools, after 4000 b.c., craftspeople in western Asia discovered that a combination of copper and tin produced bronze, a much harder and more durable metal than copper. Its [bronze’s] widespread use has led historians to call the period the Bronze Age;.. ” The pronoun “its” does not refer to Choices A, C, or D.

16. B “As wealth increased, such societies began to develop armies and to build walled cities.”

Choices A, C, and D may be logical, but they are not mentioned and may not be concluded from information in the passage.

17. C In this passage, hardly is a synonym for “barely.”

18. D Because the author states that from Neolithic towns “more complex human societies emerged,” it may be concluded that they are mentioned to contrast them with the civilizations that evolved. Choice A is not correct because a Neolithic town does not qualify as a civilization. Choice B is not correct because writing systems were not part of Neolithic settlements. Choice C is not correct because Neolithic settlements were referred to as villages, and no argument was made for the classification.

19. B “… a new social structure … [included] kings and an upper class … free people … and a

class of slaves.” Choice A is not correct because it does not include free people. Choice C is not correct because it does not include free people. Choice D is not mentioned and may not be concluded from information in the passage. The new structure described is based on economics, not on education.

20. A … the majority paraphrases “most of which.”

21. A In this passage, fundamental is a synonym for “crucial.”

22. B In this passage, important is a synonym for “prominent.” Context comes from the word “monumental” in the same sentence.

23. B “A number of possible explanations of the beginning of civilization have been suggested.”

Choice A is not correct because scholars do not agree on one explanation. Choice C is not correct because trade routes are not mentioned in paragraph 4. Choice D is not correct because coincidence is not mentioned as one of the possible explanations.

24. C Choice A is mentioned in paragraph 4, sentence 9. Choice B is mentioned in paragraph 4, sen

tence 8. Choice D is mentioned in paragraph 4, sentence 6.

25. B A rhetorical question is a question that is asked and answered by the same speaker. Response

is a transitional device that connects the insert sentence with the previous rhetorical question. Choices A, C, and D are not correct because the pronoun “they” in the insert sentence does not refer to plural nouns in the previous sentence.

26. B, E, F summarize the passage. Choice A is true, but it is a minor point that is mentioned as an

example of the characteristics of a class structure. Choice C may be one of the architectural structures built, but it is not specified. Choice D is true of Mesopotamia and Egypt, but is not developed as a major point.

Reading 3 “Life in Our Solar System”

27. D In this passage, naturally is a synonym for “automatically.” Context comes from the prefix auto,

which means “self and the root matic, which is found in many words that refer to “machines.”

28. C “The moon is airless, and although some data suggest ice frozen in the soil at its poles, it [the moon] has never had liquid water on its surface.” The pronoun “it” does not refer to Choices A, B, or D.

29. A “Venus has some traces of water vapor in its atmosphere, but it is much too hot for liquid water to survive.” Choice B is not correct because the water transformed to vapor, not ice. Ice refers to our moon, not to Venus. Choice C is not correct because the lakes or oceans evaporated auicklv. Choice D is not correct because the airless atmosphere refers to Mercury, not to Venus.

30. C In this passage, constant is a synonym for “stable.” Context comes from the contrast with “evolve” later in the sentence.

31. B Because Jupiter’s moon is used as an example of satellites of Jovian planets, it must be concluded that Jupiter is a Jovian planet. Choice A is not correct because the satellites, not the planets, have conditions that support life. Choice C is not correct because the size of Europa is not mentioned. Choice D is not correct because the author draws no conclusion about the change of orbits. If orbits were modified, then a conclusion could be drawn about whether Europa was frozen.

32. C “.. . life there may be unlikely because of the temperature. The surface of Titan is a deadly -179°C.” Choice A is not correct because there are oceans of liquid methane and ethane. Choice B is true, but it is not the reason why life would be improbable. Choice D is not correct because it is the temperature, not the atmosphere, that is low.

33. B . ..the evidence did not demonstrate paraphrases “the results did not confirm.”

34. C In this passage, begun is a synonym for “originated.” Context comes from the root origin, which means “beginning.”

35. C “ALH84001 is important because a team of scientists studied it and announced in 1996 that it contained chemical and physical traces of ancient life on Mars.” Choice A is not correct because the meteorite was found closer to twenty years ago. Choice B is not correct because the impact on Mars was not recent. Choice D is true, but it is not the reason why the author mentions the meteorite.

36. B “… conclusive evidence [of life] may have to wait until a geologist in a space suit can … open rocks … searching for fossils.” Choice A is not correct because a geologist will travel in the spacecraft. Choice C is not correct because pictures are not mentioned as a way to confirm the existence of life on Mars. Choice D is not correct because the present conditions could be studied in a variety of ways, but conclusive evidence depends on a manned flight that will allow a geologist to study the physical evidence.

37. C “We are left to conclude that, so far as we know, our solar.system is bare of life except for Earth.” Choice A is not correct because the author states that our solar system is bare of life. Choice B is not correct because the evidence is not encouraging. Choice D is not correct because the search takes us to other planetary systems.

38. C Reference is a transitional device that connects the insert sentence with the previous sentence. Such periods refers to “some points in history.” Choices A, B, and D are not correct because such periods does not refer to phrases in the sentences that precede the insert options.

39. F, D, B summarize the passage. Choice A is a minor point that supports the major point in Choice B. Choice C is a minor point that supports the major point in Choice D. Choice E is not true because the author entertains the possibility of life based on something other than carbor chemistry.

Click to rate this post!

TOEFL IBT Reading Practice Test 15 Solution & Explanation

TOEFL IBT Reading Practice Test 15 Solution & Explanation

Solution & Explanation for TOEFL iBT Reading Practice Test 15 ( From Barron’s TOEFL’S iBT)

Reading 1 “Resources and Industrialism in Canada”

1. B “The building of the Temiskaming and Northern Ontario Railway led to the discovery of rich silver deposits.” Choices A, C, and D are true, but they do not relate to the main point in paragraph 1, the resources in the western frontier.

2. C . Sudbury became the world’s largest nickel producer.” Choice A is not correct because it is not mentioned directly in the paragraph. Choice B is not correct because Sudbury was a supplier, not a market for metals. Choice D is not correct because Sudbury is in Ontario, not in the Klondike.

3. D In this passage, lasting is a synonym for “enduring.”

4. B “… the federal government created the Yukon Territory… in an effort to ward off the prospect of annexation to Alaska.” Choice A is not correct because fortune-seekers were flocking there already. Choice C is not correct because the tales of lawlessness were told in popular fiction, but no effort to establish law and order was mentioned. Choice D is not correct because the legality of the mining claims was not mentioned.

5. D In this passage, formerly is a synonym for “previously.” Context comes from the reference to “unsettled” for an area that was increasing in population.

6. A “.. . the tales [of the Klondike strike] . . . were immortalized through … the poetic verses of Robert W. Service.” Choices B and C may have been true, but they were not mentioned in connection with the poetry of Robert Service. Choice D is not correct because the creation of the Yukon Territory, not the poetry, prevented the Klondike’s annexation to Alaska.

7. B Choice A is mentioned in paragraph 3, sentence 2. Choice C is mentioned in paragraph 3, sentence 5. Choice D is mentioned in paragraph 3, sentence 4.

8. D In this passage. Moreover is a synonym for “Furthermore.” Context comes from the addition of another way that the forest and water resources were exploited.

9. C Federal taxes paraphrases “the high tariff policies” and cheaper imported goods paraphrases “lower-priced foreign manufactured goods.” . . . protecting domestic industries paraphrases “protecting existing industries” and supporting new businesses paraphrases “encouraging the creation of new enterprises.”

10. C “To climb the tariff wall, large American industrial firms opened branches in Canada, and the governments of Ontario and Quebec aggressively urged them [American industrial firms] on by offering bonuses, subsidies, and guarantees to locate new plants.” The pronoun “them” does not refer to Choices A, B, or D.

11. A “… the governments of Ontario and Quebec… [offered] bonuses, subsidies, and guarantees to locate new plants within their borders.” Choice B is true, but it is not the reason why British and American businesses opened affiliates. Choice C is not correct because the consumers in western Canada were eager to buy goods from eastern and central Canada, not from abroad. Choice D is not correct because British investors contributed to the construction of urban infrastructure.

12. C Vocabulary reference is a transitional device that connects the insert sentence with the previous sentence. The connection is “Railway construction” and “discoveries of gold, silver, copper, lead, and zinc” to “The building of the … Railway” and “rich silver deposits.”

13. C, E, F summarize the passage. Choices A and B are minor points that support major point C.

Choice D is a minor point that supports major point E.

Reading 2 “Looking at Theatre History”

14. C . the buildings in later periods became sources of stone for other projects and what remains is usually broken and scattered.” Choice A is not correct because other theatres have been identified and many of them have been excavated. Choice B is not correct because the archeologists were not the ones who broke the stones. Choice D is not correct because concrete was not mentioned as construction material during early periods. The word “concrete” in the passage means “true” or “verifiable” in reference to “evidence.”

15. B “… many pieces are irrevocably lost.” Choice A is not correct because drawings are conjectural. Choice C is not correct because the number of skenes that archeologists have excavated is not specified. Choice D is not correct because excavations did not begin until the late 1800s, not the early 1800s.

16. B In this passage, important is a synonym for “primary.” Context comes from the phrase, “most concrete evidence.”

17. D In this passage, exact is a synonym for “precise.” Context comes from the contrast with the word “conjectural” in the same sentence.

18. A . the myths on which dramatists drew were known to everyone, including vase painters, who might well depict the same subjects as dramatists.” Choice B is not correct because reproductions were not mentioned. Choice C is not correct because the qualifications of scholars were not discussed. Choice D is not correct because thousands of vases have survived.

19. B In this passage, debated is a synonym for “controversial.” Context comes from the phrases “easy to misinterpret” and “questionable assumption” in later sentences.

20. D Because the author refers to the archeological evidence in vase paintings as “controversial,” it must be concluded that there is disagreement among scholars. Choice A is not correct because the oldest surviving manuscripts date from 1500 years after they were first performed. Choice B is not correct because they are easy to misinterpret. Choice C is not correct because the author does not mention the condition of the vases.

21. B “… each time a text was copied, there were new possibilities for errors.” Choice A is not correct because the problem of sources was identified for archeological findings, not written evidence. Choices C and D are not mentioned as problems for written evidence.

22. D “… these characters [women] often seem victims of their own powerlessness.” Choice A is not correct because many plays featured strong female characters. Choice B is not correct because some critics have seen these plays [with women as victims] as rationalizations by the male-dominated culture and other critics have seen them as an attempt to examine this aspect of the culture. Choice C is not correct because plays featured numerous choruses of women.

23. B “… the majority of written references to Greek theatre date from several hundred years after the events they report. The writers seldom mention their sources of evidence, and thus we do not know what credence to give them [the sources].” The pronoun “them” does not refer to Choices A, C, or D.

24. A “… historical treatment of Greek theatre is something like assembling a jigsaw puzzle of which many pieces are missing.” The reference to “missing pieces” is an analogy to the partial evidence for Greek theatre. Choice B is not correct because no comparison is made between written references and the paintings in paragraph 4. Choice C is not correct because the author does not use words and phrases that suggest justification. Choice D is not correct because the last sentence is a summary of the reading passage, not an opening sentence for a new topic.

25. C Vocabulary reference and contrast are two transitional devices that connect the previous and following sentences to the insert sentence. The connection is “theatres … have been excavated” in the previous sentence and These excavations” in the insert sentence as well as the contrast with “Nevertheless, they” [the theatres or excavations of theatres] in the following sentence.

26. B, D, C summarize the passage. Choice A is a minor point that supports major point D. Choice E is a minor point that supports major point C. Choice F is reasonable, but it is not mentioned in the passage.

Click here to continue

Click to rate this post!

TOEFL IBT Reading Practice Test 14 Solution & Explanation

TOEFL IBT Reading Practice Test 14

SOLUTION & EXPLANATION FOR TOEFL IBT READING PRACTICE TEST 14 ( FROM BARRON’S TOEFL IBT)

Reading 1 “Beowulf”

1. C “…Beowulfwas written by an anonymous [author unknown] Englishman in Old English.”

Choice A is not correct because it is one of four surviving manuscripts. Choice B is not correct because it was written in old English about Germanic characters. Choice D is not correct because scholars do not know if it is the sole surviving epic from about a.d. 1000.

2. B “Although Beowulf was written by an anonymous Englishman in Old English, the tale takes place in that part of Scandinavia from which [that part of Scandinavia] Germanic tribes emigrated to England.”

3. A “Iron was accessible everywhere in Scandinavia, usually in the form of ‘bog iron’ found in the layers of peat in peat bogs.” Choice B is not correct because the author had already stated that the best swords had iron or iron-edged blades. Choice C is not correct because the Celts taught the Northmen how to use the materials, but they did not provide the bog iron. Choice D is not correct because the bog iron does not relate to the date, although 500 b.c. is mentioned as the time when the Northmen learned how to forge iron.TOEFL IBT Reading Practice Test 14

4. A Society in Anglo-Saxon England paraphrases “Anglo-Saxon society.” … both advanced para phrases “neither primitive,” and cultured paraphrases “nor uncultured.” Two negatives [nor and -un] produce an affirmative meaning.

5. B In this passage, rare is a synonym for “unique.” Context comes from the reference to the “sole surviving epic” in the beginning of the same sentence.

6. B “… the original manuscript was probably lost during the ninth century…. in which the Danes destroyed the Anglo-Saxon monasteries and their great libraries.” Choice A is true but it is not the reason that scholars believe the original manuscript was lost. Choice C is not correct because the Danes were invaders, not poets. Choice D is not correct because the location of the discovery is not mentioned, although the author may have been a monk.

7. D “Although the Beowulf manuscript was written in about a.d. 1000, it was not discovered until the seventeenth century.” Choice A is not correct because the first century was the date the manuscript was written, not discovered. Choice B is not correct because the ninth century was the date when the original manuscript may have been lost. Choice C is not correct because some scholars think that the manuscript was written in the eleventh century.

8. A Because the word “apparently” means “appearing to be so,” the author is expressing doubt about the information that follows, “… [the Beowulf poet] was a Christian.” Choice B is not correct because the word “obviously” would be used. Choice C is not correct because the phrases “for example” or “for instance” would signal an example. Choice D is not correct because evidence would not be presented as “appearing to be so.”

9. A “.. . Beowulf is a very appealing hero … Like Hercules.” Choice B is not correct because a fight with a dragon is mentioned in reference to Beowulf but not to Hercules. Choice C is not correct because the Danish hero’s welcome is the only reference to a speech, and it was jealous, not inspiring. Choice D is not correct because the time period for the life of Hercules is not mentioned.

10. B In this passage, demonstrates is a synonym for “exhibits.”

11. C In this passage, refuse is a synonym for “reject.” Context comes from the contrast with “accept” in the previous sentence.

12. C Addition is a transitional device that connects the insert sentence with the previous sentence.

Moreover signals that additional, related information will follow. “. . . they [scholars] disagree” refers to “Scholars do not know” in the previous sentence.

13. E, D, F summarize the passage. Choice A is true, but it is a minor point that establishes the time period for the poem and refers to major point D. Choice B is true, but it is a detail that refers to major point E and explains why there may be only one manuscript. Choice C is not clear from the information in the passage.

Reading 2 “Thermoregulation”

14. A “The most basic mechanism [for maintenance of warm body temperature] is the high meta bolic rate.” Choices B, C, and D are all ways to maintain body temperature, but they are not the most fundamental adaptation.

15. D “In some mammals, certain hormones can cause mitochondria to increase their metabolic activity and produce heat instead of ATP. This non shivering thermogenesis (NST). . . .” Choice A is not correct because thermogenesis is the activity that generates heat, not the heat loss. Choice B is not correct because brown fat is one example of a more generalized process. Choice C is not correct because thermogenesis is a response to the environment to maintain the health of the animal, not a process that maintains the environment.

16. B A passive grammatical structure in the passage is paraphrased by an active grammatical structure in the answer choice.

17. D In this passage, smallest is a synonym for “minimal.”

18. B “For example, heat loss from a human is reduced when arms and legs cool.” Choice A is not correct because goose bumps, not heat loss in the extremities, is a vestige of our evolution. Choice C is not correct because no direct comparisons of -these processes are made in the paragraph. Choice D is not correct because the types of insulation are mentioned before the concept of vasodilatation and vasoconstriction are introduced.

19. D In this passage, control is a synonym for “regulate.” Context comes from the reference to “tem perature differences” at the end of the same sentence.

20. B “The loss of heat to water occurs 50 to 100 times more rapidly than heat loss to air.” Choice A is not correct because hair loses insulating power when wet, but the evolution of marine animals is not mentioned. Choice C is not correct because dry hair insulates better than wet hair. Choice D is not correct because there are land animals that are of similar size.

21. D “… marine mammals maintain body core temperatures of about 36-38°C with metabolic rates about the same as those [metabolic rates] of land mammals of similar size.”

22. A “. . . capable of astonishing feats of thermoregulation. For example, small birds called chickadees … hold body temperature nearly constant.” Choice B te not correct because the food supply supports thermoregulation, which is the main point of the example. Choice C is not correct because chickadees are capable of astonishing feats of thermoregulation. Choice D is not correct because the reason for heat production in animals is explained before the example of the chickadee.

23. D Choice A is mentioned in paragraph 6, sentence 7. Choice B is mentioned in paragraph 6, sentence 8. Choice C is mentioned in paragraph 6, sentences 3 and 4.

24. B In this passage, improve is a synonym for “enhance.” Context comes from the reference to “promote” in the previous sentence.

25. A Reference is a transitional device that connects the insert sentence with the previous sentence. “… a layer of fur or feathers” and “how much still air the layer [of fur or feathers] traps” in the insert sentence refers to “… fur or feathers” and “a thicker layer of air” in the previous sentence.

26. E, C, F summarize the passage. Choice A is a minor point that supports major point C. Choice B is true but it is not mentioned in the passage. Choice D is a minor point that supports major point F.

Reading 3 “Social Readjustment Scales”

27. D “Overall, these studies have shown that people with higher scores on the SRRS tend to be more vulnerable to many kinds of physical illness ” Choice A is not correct because a person with a higher score will experience more, not less, stress. Choice B is not correct because the numerical values for major problems are not identified, and a score of 30 does not have meaning unless it is compared with a higher or lower score. Choice C is not correct because the effects of positive or negative change are not mentioned in the first two paragraphs.

28. C “… the desirability qf events affects adaptational outcomes more than the amount of change that they [events] require.”

29. D In this passage, different is a synonym for “diverse.”

30. C “. .. divorce may deserve a stress value of 73 for most people, a particular person’s divorce might generate much less stress and merit a value of only 25.” Choice A is not correct because a particular person is compared with most people. Choice B is not correct because the serious nature of divorce is not mentioned. Choice D is not correct because the numerical value of 73 for most people is questioned.

31. A “… what qualifies as trouble with the boss’? Should you check that because you’re sick and tired of your supervisor? What constitutes a ‘change in living conditions’? Does your purchase of a great new sound system qualify?” Choice B is not correct because the author does not offer examples of responses to the questions posed. Choice C is not correct because options for scores are not provided in paragraph 5. Choice D is not correct because the author suggests that people do not respond consistently but whether they respond carefully is not mentioned.

32. B . subjects’ neuroticism affects both their responses to stress scales and their self-reports of health problems.” Choice A is not correct because they recall more symptoms, but they are not ill more often. Choice C is not correct because they recall more stress, but they do not necessarily suffer more actual stress. Choice D is not correct because the effects of neuroticism obscures the meaning of the scores that are recorded.

33. C In this passage, arranged is a synonym for “assembled.”

34. C In this passage, related is a synonym for “relevant.”

35. B “… dropping the normative weights and replacing them with personally assigned weightings.”

Choice A is not correct because long-term consequences are not included in positive, negative, and total change scores. Choice C is not correct because the differences in people reflect their appraisal of stress, not how they handle stress. Choice D is not correct because normative weighting is replaced by personally assigned weightings.

36. C In paragraph 1, the authors state that the SRRS “… assigns numerical values.” Choices A and B are not correct because they are mentioned in paragraph 10 in reference to the LES, not the SRRS. Choice D is not correct because recalling events from one year ago is a problem on the SRRS.

37. A “The LES deals with the failure of the SRRS to sample the full domain of stressful events.”

Choice B is not correct because the author explains several ways that the LES deals with the failure of the SRRS. Choice C is not correct because it has been used in thousands of studies by researchers all over the world. Choice D is not correct because the LES, not the SRRS, has a special section for students.

38. B Reference is a transitional device that connects the insert sentence with the previous sentence. “This sum” in the insert sentence refers to the phrase “adds up the numbers” in the previous sentence.

39. SRRS: A, H, I LES: B, D, E, G Not used: C, F

Click to rate this post!

TOEFL iBT Listening Practice Test 03 Solution & Transcription

TOEFL iBT Listening Practice Test 03 Solution & Transcription

TOEFL iBT Listening Practice Test 03 Solution

Listening 1 “Student on Campus”

1. B

2. C

3. A

4. C

5. B

Listening 2 “Sociology Class”

6. C

7. B, C

8. C

9. C

10. A : YES/ B: NO / C : YES

11. B

Listening 3 “Art History Class”

12. D

13. B, C

14. C

15. B

16. D

17. B

Listening 4 “Admissions Office”

18. D

19. B

20. B

21. D

22. B

Listening 5 “Anthropology Class”

23. A

24. B

25. B, C

26. B

27. A

28. B

Listening 6 “Geology Class”

29. B

30. A

31. D

32. B

33. B

34. A

Listening 7 “Library”

35. B

36. B

37. D

38. D

39. B

Listening 8 “Literature Class”

40. A

41. A

42. C

43. A

44. B

45. D

Listening 9 “General Science Class”

46. D

47. B

48. B

49. C

50. A

51. B

TOEFL iBT Listening Practice Test 03 Transcripts

LISTENING 1 “ STUDENT ON CAMPUS”

Audio Conversation

Narrator: Listen to part of a conversation on campus between two students.

Man: I wish I were as sure about my future as you seem to be. I… I realLy don’t know what I want to do after I graduate. 

Woman: Well, have you talked with a counselor over at the Office of Career Development?

Man: No…I talked to my academic advisor, though.

Woman: That’s good, but it’s really better to see someone who specializes in helping people make career decisions. You see, an academic advisor is there to help you work out your academic program. You know, figure out what your major is going to be and which courses to take and all that. But a career counselor has a lot of experience and resources to help you decide what you want to do in the work world.

Man:Did you see a career counselor?

Woman:I sure did. Last semester. I was… well, I didn’t even know what I would be good at, for a career, I mean. So I made an appointment at the Office of Career Development, and I talked with a counselor.

Man:Do you remember who it was?

Woman:Sure. It was Ruth Jackson.

Man:Oh, but since I’m interested in careers for math majors, probably I should see someone else.

Woman:Not really. Any of the counselors can help you. Look, first I took some aptitude tests and something called a … uh … I think it was called a career inventory. Anyway, I took several tests, and then the counselor gave me some ideas about different careers. I even went to some group sessions with some other students for a few weeks. Mrs. Jackson was the group leader, so, um, that’s how I met her, and then I just sort of naturally started making my appointments with her when I needed some advice.

Man: It sounds like it took a lot of time. I’m so busy already.

Woman: Well, it did take time. Probably three hours for the tests, and I think I went to maybe four group sessions, and then I saw Ruth a couple of times. I guess about nine or ten hours probably. But it was worth it.

Man: So, is that why you decided to go into library science? Because of the tests and everything?

Woman: In part But, mostly it was because of the internship. You see, I also got my internship through the Office of Career Development. And when I was working as an intern in the public library, it all sort of came together tor me. I really liked what I was doing, and I realized that I didn’t want the internship to end.

Man: And you get paid for working there in the library too, don’t you?

Woman: I get paid, and I get credit toward my degree. But even better, I have a job offer from the library where I’m doing my internship.

Man: Wow! Are you going to take it?

Woman: I think so. I have to let them know next week. If I do take the job, I’ll have to go to graduate school to get a degree in library science, but I can do that part-time while I’m working, and I had thought about graduate school anyway. So, I’m leaning toward taking the job.

Man: That’s great, Anne. I’m glad for you. So, uh, I guess I’d better make an appointment with Ruth Jackson. Maybe she can find me an internship.

Woman: Maybe.

LISTENING 2 “SOCIOLOGY CLASS”

Audio Lecture

Narrator Listen to part of a lecture in a sociology class.

Professor:

Social influence involves the changes in behavior influenced by the actions of other people. Social influence can come about for a variety of reasons, on a continuum from mere suggestion to, in the more severe form, well, to torture. How does social influence work? Well, first we must become aware of a difference between ourselves and the values or behaviors of other people. There are a great many studies of social influence that demonstrate how the presence of others can cause us to change our attitudes or actions. Studies show that people eat more when dining with others than, and I’m talking about dining out here, so they eat more in the company of others than they do when they’re alone. They also run faster when others are running with them. There’s even some interesting research on social influence among animals with similar results to… to those of human studies.

Probably one of the most interesting aspects of social influence is the pressure for conformity. Con-formity is a process by which an individual’s opinion or behavior moves toward the norms of the group. In a dassic study by Solomon Asch, seven people were shown cards with three lines drawn on them. Here’s an example:

So, they were shown the lines, and then they were asked to select the line among the three that matched the, uh … the … standard line. Here’s the standard. So there’s no question as to the comparison This has to be easy, right? Wrong. You see, Asch enlisted the cooperation of six of the seven participants in the experiment. On the first card, the six respond correctly—they . . . they identify the lines of the same length—so the seventh person, who is the only real subject in the experiment, well, the seventh person answers correctly, in agreement with the others. But on the next card, four of the cooperating participants choose an incorrect answer, but they’re in agreement, so the problem for the subject is whether to conform to the opinion of the peer group, even though the answer, uh, is in conflict with the answer that the subject knows to be correct.

So what do you think happened? Well, subjects who were tested alone made errors in answers fewer than 1 percent of the time. This was the control group. But of those tested in groups of seven, let’s see. uh, 75 percent yielded at least once to conform to a group answer that was ctearty incorrect, and on average, subjects conformed to the group in about 37 percent of the critical trials. This means that they were bring-ing their behavior into agreerrtent with group norms in … in spite of what they were seeing.

Later Asch manipulated the size of the control group … I’m sorry, the experimental group… to see whether group size would affect pressure, and it did, but probably less than you might expect. Um … groups of four demonstrated about the same results as groups of eight. Interestingly enough, a unanimous agreement by the group was more important than the number. In other words, a unanimous opinion by three exerted more pressure to conform than a majority of seven with a dissenting opinion in a group of eight.

Similar experiments have been performed in various countries, among diverse cultural groups, with, um, comparable results. Of course, people in cultures that emphasize group cooperation tended to be more willing to conform, but remember that many of the original studies were done in the United States where there’s a high value placed on individualism. In an interesting variation on the study, Abrams found that conformity is especially strong when the group is selected from among those people that the subject dearly identifies with, either because, um … they have characteristics in common or… or they know each other and interad in a peer group outside of the experimental situation.

So what does ail of this mean in the real world? Well, since group members can influence one another to conform to the opinion of the group, the group… decisions of a group, uh, may be called into question. What about decisions by political committees or parliaments? What about juries who are charged with convicting or acquitting an accused defendant? Clearfy, social influence will play a part in these critical group decisions.

Also interesting is the fad that after a decision is made by a group, there’s a tendency to solidify, and by that I mean that the group becomes even more convinced of the validity of the group opinion. Um … this may happen because individual group members who strongly support the group tend to be more popular with the group members.

Click to rate this post!

Barron’s TOEFL iBT Superpack by Pamela Sharpe, Steven J. Matthiesen

Barron's TOEFL iBT Superpack

Barron’s TOEFL iBT Superpack by Pamela Sharpe, Steven J. Matthiesen

EVERYTHING YOU NEED TO EXCEL ON THE TOEFL

Barron’s is one of the international leaders when it comes to test preparation for the SAT and TOEFL tests. Last year they have gone ahead and presented their Superpack which contains all things you need for your preparation. This is something I admire because with so many other resources you have to spend more money for things like Audio CDs and Answer Keys.

Barron's TOEFL iBT Superpack

Barron’s TOEFL iBT Superpack

Barron’s TOEFL iBT Superpack study set is with good reason one of the most popular TOEFL test prep guides.

This newly released set for 2016-2017 consists of four essential books and multimedia resources: Barron’s TOEFL iBT manual with MP3 and audio CD (for all of the book’s practice test questions, practice TOEFL tests, an additional TOEFL test with automatic scoring), Barron’s Writing for the TOEFL iBT book with MP3 CD, TOEFL Strategies and Tips with MP3 CD, and the Essential Words for the TOEFL guide to enhance your vocabulary.

The heart of this set is however Barron’s TOEFL iBT manual. This resource is notable for providing a comprehensive yet clear and accessible guide that will help you familiarize yourself with the basics of the TOEFL test and its listening, speaking, reading, and writing sections. TOEFL iBT also includes seven practice tests with detailed notes and explanations. There is also a section that outlines the key academic skills necessary to ace the test, such as note taking and synthesizing, and access to TOEFL video lessons.

With the TOEFL Superpack, studying becomes more interesting and efficient thanks to mix of books, audio and MP3 CDs, practice questions and tests, and  video lessons. This is clearly one of the best TOEFL study guides – highly recommended!

I think that this is an excellent resource for anyone, trying to crack the TOEFL, but, above all, I would strongly recommend this “Superpack” to people who want to achieve a score above 80 pts. on the TOEFL but don’t want to spend a whole fortune on preparation materials.

Download Barron’s TOEFL iBT Superpack by Pamela Sharpe, Steven J. Matthiesen

Top Customer Reviews on Amazon:

5.0 out of 5 stars : Great SUPERPACK! 

By Maria Paz on May 24, 2016

I definitely recommend this pack to anyone who’s planning to pass the TOEFL at the first shot!
It’s very complete preparation program and it gives you a lot of tips to make out of the TOEFL a success.I got it a week before taking the test, Took about 2 days on the book “Words you need to know for the TOEFL” and then I basically used the CD for performing Toefl test models to verify my time etc.If you’ve got a decent level of English don’t bother paying $1000+ TOEFL preparation courses ’cause you’ll do exactly the same that you’ll find in this book

5.0 out of 5 stars: Excellent material 

By Ricardo Brazil on August 9, 2016

I received the pack some days ago and I have already studied some lessons. This was enough to realize the material is very good: the CD’s have good quality, excellent book with clarifying information and the CD ROM contains a practice test, very useful to the students get in use with the test day. No doubt this is an excellent acquisition.

4.0 out of 5 stars: Good value but flaws in CD
By kim on February 25, 2016

I bought this product a week ago. It arrived on time, every thing was good. The book is good, the small book comes with is good but when I started doing the full test, I started to feel really annoyed. When I take the full test, it skips some questions and at the end of the test you end up with 9 unanswered questions. Moreover it shows the questions I did right as wrong. It shows the same choice as a right answer but still gives red mark telling I did wrong. So these really demotivate you, because you wanna see how well you scored but constantly facing with problems is frustrating. Still if you want to evaluate yourself with the chart ( number of right answer Vs. scaled scores) you can benefit from the material. There are 8 full practice tests, so plenty of exercise.

Click to rate this post!

TOEFL IBT Reading Practice Test 20 from Barron’s TOEFL iBT

TOEFL IBT Reading Practice Test 20 from Barron's TOEFL iBT

TOEFL IBT Reading Practice Test 20 from Barron’s TOEFL iBT

The Reading section tests your ability to understand reading passages like those in college textbooks. The passages are about 700 words in length.

This is the short format for the Reading section. On the short format, you will read three passages. After each passage, you will answer 12-14 questions about it. You may take notes while you read, but notes are not graded. You may use your notes to answer the questions. Continue reading

Click to rate this post!